Solve Simple Logarithms: log91/27

  • Thread starter matadorqk
  • Start date
  • Tags
    Logarithms
M = xlog(a)M/xlog(a)b... but x = log(b)M ... log(b)Mlog(b)M = log(b)Mlog(a)M/log(a)b... log(b)M = log(b)Mlog(a)M/log(a)b... log(b)M = log(a)M/log(a)b... log(b)M = log(a)M/log(a)b... x = log(a)M/log(a)b... x = log(a)M/log(a)b... x = log(a)M/log(a)b = log(b)M... log(b)M
  • #1
matadorqk
96
0

Homework Statement


#8: Simplify: log91/27

Homework Equations



y=logb(x)
x=b^y

The Attempt at a Solution



I have personally not tried logarithms before, but I think the problem is really easy and I am probably just over-thinking it?

I think that, using the formula's below, 9^a=1/27
So, a=1/243, so my simplified answer should be 9^1/243=1/27..
Can I simplify fractions then, so that 9^1/9=1?
I have personally no exact knowledge on what I am doing, but can someone give me some hints or basic concepts on logarithms? Thanks!

**Edit:
Ok, I might be wrong above, I found some more laws, and since loga (x/y)= logax - loga y,
I get:
log 9 (1) - log 9 (27)

since loga 1 = 0
I know have:
- log 9 (27)
Err.. now what?
I have no idea how to plot it into my calculator...
But from my formulas (first two) I get that 9^2/3=27.. but does simplifying mean turning into exponential form? :S
 
Last edited:
Physics news on Phys.org
  • #2
27 = 3^3 = 9^(3/2) ==> log_9(27) == 3/2
 
  • #3
genneth said:
27 = 3^3 = 9^(3/2) ==> log_9(27) == 3/2

Huh? I'm pretty sure its 2/3... maybe I am wrong but could you further explain yourself?
 
  • #4
log(base9) (1/27) = x

9^x = (1/27)

x = log(1/27)/log(9)
 
  • #5
rocophysics said:
log(base9) (1/27) = x

9^x = (1/27)

x = log(1/27)/log(9)

So log (1/27) / log (9)

log (1/27) = - log 27 / log 9
Can I simplify to - log 3 / log (1)
So - log 3 / ..0?

Hmm, so I can't simplify or my answer is undefined...

Is my answer -log 27 / log 9..
Im really lost here.

How about this:
Since log (base a) (a) = 1, does log (base a) (3a) = 3? If so, log (base 9) (27) = 3?
 
  • #6
log(base9) (1/27) = x

9^x = (1/27) in exponential form ... since it said to simplify, i solved sorry

type in your calculator log(1/27)/log(9) and plug into x, 9^(-1.5) = 1/27
 
  • #7
rocophysics said:
log(base9) (1/27) = x

9^x = (1/27) in exponential form ... since it said to simplify, i solved sorry

No problem. So, 9^x= 1/27 is simplified?..

*bangs head against wall* ( I knew it wasn't that complicated, :P)
 
  • #8
so the general rule, and with the relevant equations you gave should have been enough information for you to solve it

anyways, the way i learned to convert/simplify/solve logs is by how you read logs

log(base) (y) = x

the base must be raised to some power x, to get y

logs are just inverses, one-to-one functions
 
  • #9
rocophysics said:
so the general rule, and with the relevant equations you gave should have been enough information for you to solve it

anyways, the way i learned to convert/simplify/solve logs is by how you read logs

log(base) (y) = x

the base must be raised to some power x, to get y

logs are just inverses, one-to-one functions

Thanks a lot for your help, ill go on to the next log problem :) hehe, I guess it wasn't so complicated, I just really didn't know what they ment by "simplify" but now I will assume that simplifying a log solely involves getting rid of the 'log'. Again, thanks.
 
  • #10
matadorqk said:
Thanks a lot for your help, ill go on to the next log problem :) hehe, I guess it wasn't so complicated, I just really didn't know what they ment by "simplify" but now I will assume that simplifying a log solely involves getting rid of the 'log'. Again, thanks.
i just reviewed logs Monday so it's fresh on my mind!

some can be a real b tho :) have fun
 
  • #11
Actually simply means simplify until you cannot simplify anymore. If you can reach a solution that is how far you need to go, like in this case.

Here's how I did it (since you already have the answer :-/):

[tex]log_9(\frac{1}{27}) = \frac{log_{3}\frac{1}{27}}{log_{3}9}[/tex]

[tex]\frac{log_{3}\frac{1}{27}}{log_{3}9} = \frac{log_{3}3^{-3}}{log_{3}3^2}[/tex]

[tex]\frac{log_{3}3^{-3}}{log_{3}3^2} = \frac{-3}{2}[/tex]
 
  • #12
matadorqk said:
Huh? I'm pretty sure its 2/3... maybe I am wrong but could you further explain yourself?

[tex]27 = 9 * 3 = 9 * 9^{1/2} = 9^{1 + 1/2} = 9^{3/2}[/tex]

[tex]1/27 = 27^{-1} = (9^{3/2})^{-1} = 9^{-3/2}[/tex]
 
Last edited:
  • #13
Feldoh said:
Actually simply means simplify until you cannot simplify anymore. If you can reach a solution that is how far you need to go, like in this case.

Here's how I did it (since you already have the answer :-/):

[tex]log_9(\frac{1}{27}) = \frac{log_{3}\frac{1}{27}}{log_{3}9}[/tex]

[tex]\frac{log_{3}\frac{1}{27}}{log_{3}9} = \frac{log_{3}3^{-3}}{log_{3}3^2}[/tex]

[tex]\frac{log_{3}3^{-3}}{log_{3}3^2} = \frac{-3}{2}[/tex]

Wow... Thanks a lot for that! That explains a lot :P
 
  • #14
Feldoh said:
[tex]log_9(\frac{1}{27}) = \frac{log_{3}\frac{1}{27}}{log_{3}9}[/tex]

[tex]\frac{log_{3}\frac{1}{27}}{log_{3}9} = \frac{log_{3}3^{-3}}{log_{3}3^2}[/tex]

[tex]\frac{log_{3}3^{-3}}{log_{3}3^2} = \frac{-3}{2}[/tex]

Umm, may I ask how you get from [tex]log_9(\frac{1}{27}) = \frac{log_{3}\frac{1}{27}}{log_{3}9}[/tex]

Is there a formula?
Like...: log(base a) (b) = log (base (square root of a)) (b) / log (base (square root of a)


You kind of make sense, but I want to know how to implement it in any problem..
I hope you don't mind explaining after you solved it...
 
  • #15
matadorqk said:
Umm, may I ask how you get from [tex]log_9(\frac{1}{27}) = \frac{log_{3}\frac{1}{27}}{log_{3}9}[/tex]

Is there a formula?
Like...: log(base a) (b) = log (base (square root of a)) (b) / log (base (square root of a)You kind of make sense, but I want to know how to implement it in any problem..
I hope you don't mind explaining after you solved it...

Change of base is what it is, most people don't realize you can change to any base and not just base 10.

[tex]log_{a}b = \frac {log_{n}b}{log_{n}a}[/tex] where n is an integer and is the new base.
 
Last edited:
  • #16
Feldoh said:
Change of base is what it is, most people don't realize you can change to any base and not just base 10.

[tex]log_{a}b = \frac {log_{n}b}{log_{n}a}[/tex] where n is an integer and is the new base.

Umm, but leaving n as an integer doesn't that give room for n=any integer while a and b don't change? I am going to analyze this and ill get back to you.
 
  • #17
change-of-base formula is ... log(b)M=log(a)M/log(a)b where log(a) = log(base10)

let's start off with the Proof

Proof, let x = log(b)M ... in exponential form b^x = M

1) b^x = M

2) take the log(a) of both sides ... log(a)b^x = log(a)M

3) solve for x ... x = log(a)M/log(a)b

4) if x = log(b)M ... (step 3) log(b)M = log(a)M/log(a)b

... log(a) can be anything, it doesn't have to be log(base10)
 
Last edited:
  • #18
rocophysics said:
change-of-base formula is ... log(b)M=log(a)M/log(a)b where log(a) = log(base10)

let's start off with the Proof

Proof, let x = log(b)M ... in exponential form b^x = M

1) b^x = M

2) take the log(a) of both sides ... log(a)b^x = log(a)M

3) solve for x ... x = log(a)M/log(a)b

4) if x = log(b)M ... (step 3) log(b)M = log(a)M/log(a)b

... log(a) can be anything, it doesn't have to be log(base10)

so... if my answer is log(3) 3^-3 / log (3) 3^2 = -3/2...
Can it also be log (5) 3^-3 / log (5) 3^2 = -3/2.. ?

Or does the new base have to be related to the original base? This is what has me confused.. How do you define the new base, and what effect does this have on the other terms of the equation?

This would be because even though even though the numbers may be larger/smaller, the ratio is the same? I hope I am right this time, I think I am getting this logs thing :)
 
Last edited:
  • #19
i think so ... lol, but I'm no expert.
 
  • #20
Thanks, ill re-check with my teacher tommorow. Anyways, I got stuck again.

Homework Statement


Convert [tex]\frac{8}{\log_{5}9}[/tex] to an expression of the form [tex]a\log_{3}b[/tex]
Ok that's the problem.

Homework Equations


#1: [tex]y=\log_{a}(x) [/tex]
#2: [tex] x=a^y [/tex]

The Attempt at a Solution


**Im working on it as you read :) bare with me and I'll give you my attempt hehe
 
Last edited:
  • #21
hmm ... let me evaluate, i see that 8 can be re-written as 2^3 & 9 as 3^2 ... still thinking

... kicking my ass! hehe

is this correctly typed in?
 
Last edited:
  • #22
Convert: [tex]\log_{5}9}[/tex] to log base 3, using the method discussed in this thread...
 
  • #23
meh, I just figure the exponent must be negative if it is to produce a fractional result. So, then the problem just reduces to a matter of manipulating 9 to get 27. taking the square root of 9 yields 3 and cubing 3 yields 27. So, the exponent would need to be -2/3
 
  • #24
learningphysics said:
Convert: [tex]\log_{5}9}[/tex] to log base 3, using the method discussed in this thread...

Great!
So, 2^3 over: [tex]\frac{\log_{3}3^2}{\log_{3}5}?[/tex]

leright said:
meh, I just figure the exponent must be negative if it is to produce a fractional result. So, then the problem just reduces to a matter of manipulating 9 to get 27. taking the square root of 9 yields 3 and cubing 3 yields 27. So, the exponent would need to be -2/3

As much as that seems simpler and faster, if I don't learn the method to actually do it for any problem instead of solving 1, I'll have problems with similar probs. And um, I think we solved that one already, its actually -3/2
 
Last edited:
  • #25
matadorqk said:
Great!
So, 2^3 over: [tex]\frac{\log_{3}3^2}{\log_{3}5}?[/tex]

Sorry, yes. 8 over that. And that's it. So I get 4log_3{5}
 
Last edited:
  • #26
learningphysics said:
Sorry, yes. 8 over that. And that's it. So I get 4log_3{5}

Err.. big jump there :P, What do I do when I have a.. three layer fraction?
..nvm.. Lemme figure this out
 
  • #27
matadorqk said:
Err.. big jump there :P, What do I do when I have a.. three layer fraction?
say a/b/c= ca/b?

[tex]\frac{a}{(\frac{b}{c})} = \frac{ac}{b}[/tex]

but: [tex]\frac{(\frac{a}{b})}{c}=\frac{a}{bc}[/tex]
 
  • #28
learningphysics said:
[tex]\frac{a}{(\frac{b}{c})} = \frac{ac}{b}[/tex]

but: [tex]\frac{(\frac{a}{b})}{c}=\frac{a}{bc}[/tex]

I used this formula and the one above,
[tex]\log_{a}x^y=y\log_{a}x[/tex]

[tex]\frac{8(\log_{3}5)}{2\log_{3}3}[/tex]

Right?

But then I get
[tex]4\log_{3}\frac{5}{3}[/tex]
 
  • #29
matadorqk said:
I used this formula and the one above,
[tex]\log_{a}x^y=y\log_{a}x[/tex]

[tex]\frac{8(\log_{3}5)}{2\log_{3}3}[/tex]

Right?
That's right.

But then I get
[tex]4\log_{3}\frac{5}{3}[/tex]

this is wrong. (log_a{x})/(log_a{y}) is not equal to log_a{x/y}. what is log_3{3}?
 
  • #30
learningphysics said:
That's right.



this is wrong. (log_a{x})/(log_a{y}) is not equal to log_a{x/y}. what is log_3{3}?

Ugh right! It equals 1!

So there we have it. [tex]\frac{8}{2}(log_{3}5)[/tex] So: [tex] 4\log_{3}5 [/tex] Yay!

Improvement!
 
  • #31
Thanks evrn!

Homework Statement



Express as a single logarithm 2lnx + ln (x-1) - ln (x-2)

Homework Equations



None used (Basic Algebra)

The Attempt at a Solution



So, let's substitute ln for a, to make it simpler
2ax +a (x-1) - a (x-2)
2ax +ax - a - ax +2a
2ax - a +2a
2ax+a
2lnx+ln
ln (2x+1)

I think that's correct, I'd appreciate some re-check just in case though.

That's the last one, thanks everyone for the help!
 
  • #32
matadorqk said:

Homework Statement



Express as a single logarithm 2lnx + ln (x-1) - ln (x-2)

Homework Equations



None used (Basic Algebra)

The Attempt at a Solution



So, let's substitute ln for a, to make it simpler
2ax +a (x-1) - a (x-2)
2ax +ax - a - ax +2a
2ax - a +2a
2ax+a
2lnx+ln
ln (2x+1)

I think that's correct, I'd appreciate some re-check just in case though.

That's the last one, thanks everyone for the help!

You can't substitute for ln... ln is not a variable. it's a function... You could substitute for something like ln(a) which is a value, or ln(x)... but not just ln...
 
  • #33
2lnx = ln(x^2)

ln(x-1) - ln(x-2) = ln[(x-1)/(x-2)]

ln(x^2) + ln[(x-1)/(x-2)] = ln[(x^3-x^2)/(x-2)]
 
  • #34
learningphysics said:
You can't substitute for ln... ln is not a variable. it's a function... You could substitute for something like ln(a) which is a value, or ln(x)... but not just ln...

Uh oh.. Ok let's restart again then.

So ln= [tex] log_{e}x [/tex]

Ok, ill go from there and get back to you

**Or take roco's idea, a head start :)
 
  • #35
matadorqk said:
**Or take roco's idea, a head start :)
nooo!
 

Similar threads

  • Precalculus Mathematics Homework Help
Replies
4
Views
1K
  • Precalculus Mathematics Homework Help
Replies
12
Views
2K
  • Precalculus Mathematics Homework Help
Replies
23
Views
771
  • Precalculus Mathematics Homework Help
Replies
7
Views
1K
  • Precalculus Mathematics Homework Help
Replies
5
Views
1K
  • Precalculus Mathematics Homework Help
Replies
1
Views
568
  • Precalculus Mathematics Homework Help
Replies
1
Views
860
  • Precalculus Mathematics Homework Help
Replies
10
Views
642
  • Precalculus Mathematics Homework Help
Replies
2
Views
777
  • Precalculus Mathematics Homework Help
Replies
4
Views
780
Back
Top